Đến nội dung

Hình ảnh

$f(mn)=f(m)f(n)$

- - - - - 100hamso

  • Please log in to reply
Chủ đề này có 2 trả lời

#1
namcpnh

namcpnh

    Red Devil

  • Hiệp sỹ
  • 1153 Bài viết

Bài 9 :Tìm tất cả các hàm số $f:\mathbb{Z}^+\rightarrow \mathbb{Z}^+$ thỏa mãn :

 

1)$f(2)=2$

 

2)$f(mn)=f(m)f(n)$

 

3)$f(n+1)\geq f(n)$

 

Bài giải :

 

Từ $(2)$ bằng qui nạp chứng minh được $f(n^k)=(f(n))^k$ với $k \in \mathbb{N}$ và $k \geq 2$
Dễ có $f(2^k)=(f(2))^k=2^k$
Cho $m=1$ có $f(m)=f(m)f(1) \Rightarrow f(1)=1$
Từ $(3)$ giả sử $f(n+1)=f(n)$ với $n \geq 2$
Ta có $f((n+1)^t)=(f(n+1))^t=(f(n))^t=f(n^t) \Rightarrow f(m)=c, (n+1)^t \geq m \geq n^t$ $(*)$
Ta thấy tồn tại $c,t$ thỏa $(n+1)^t >2^{c+1}>2^c>n^t$ mà $2^{c+1}=f(2^{c+1}) \neq f(2^c)=2^c$ mâu thuẫn với $(*)$
Vậy $f(n+1)>f(n)$ $(**)$
Ta có $f(2^{k+1})-f(2^k)=2^{k+1}-2^k$ nên theo $(**) \Rightarrow f(n)=n$ với $2^{k+1} \geq n \geq 2^k$
Từ đó chứng minh được $f(n)=n$ với $n \in \mathbb{Z^+}$
Vậy hàm thỏa mãn đề là $f(n)=n$


Bài viết đã được chỉnh sửa nội dung bởi namcpnh: 19-05-2013 - 18:16

Cùng chung sức làm chuyên đề hay cho diễn đàn tại :

Dãy số-giới hạn, Đa thức , Hình học , Phương trình hàm , PT-HPT-BPT , Số học.

Wolframalpha đây


#2
Idie9xx

Idie9xx

    Sĩ quan

  • Thành viên
  • 319 Bài viết

Bài 9 :Tìm tất cả các hàm số $f:\mathbb{Z}^+\rightarrow \mathbb{Z}^+$ thỏa mãn :

 

1)$f(2)=2$

 

2)$f(mn)=f(m)f(n)$

 

3)$f(n+1)\geq f(n)$

Từ $(2)$ bằng qui nạp chứng minh được $f(n^k)=(f(n))^k$ với $k \in \mathbb{N}$ và $k \geq 2$

Dễ có $f(2^k)=(f(2))^k=2^k$

Cho $m=1$ có $f(m)=f(m)f(1) \Rightarrow f(1)=1$ 

Từ $(3)$ giả sử $f(n+1)=f(n)$ với $n \geq 2$

Ta có $f((n+1)^t)=(f(n+1))^t=(f(n))^t=f(n^t) \Rightarrow f(m)=c, (n+1)^t \geq m \geq n^t$ $(*)$

Ta thấy tồn tại $c,t$ thỏa $(n+1)^t >2^{c+1}>2^c>n^t$ mà $2^{c+1}=f(2^{c+1}) \neq f(2^c)=2^c$ mâu thuẫn với $(*)$

Vậy $f(n+1)>f(n)$ $(**)$

Ta có $f(2^{k+1})-f(2^k)=2^{k+1}-2^k$ nên theo $(**) \Rightarrow f(n)=n$ với $2^{k+1} \geq n \geq 2^k$

Từ đó chứng minh được $f(n)=n$ với $n \in \mathbb{Z^+}$

Vậy hàm thỏa mãn đề là $f(n)=n$ >:)


Bài viết đã được chỉnh sửa nội dung bởi Idie9xx: 19-05-2013 - 13:05

$\large \circ \ast R_f\cdot Q_r\cdot 1080\ast \circ$

#3
namcpnh

namcpnh

    Red Devil

  • Hiệp sỹ
  • 1153 Bài viết

Bài giải của Idie9xx hợp lí rồi , các bạn vào cho ý kiến :D .


Cùng chung sức làm chuyên đề hay cho diễn đàn tại :

Dãy số-giới hạn, Đa thức , Hình học , Phương trình hàm , PT-HPT-BPT , Số học.

Wolframalpha đây






Được gắn nhãn với một hoặc nhiều trong số những từ khóa sau: 100hamso

2 người đang xem chủ đề

0 thành viên, 2 khách, 0 thành viên ẩn danh